LSAT and Law School Admissions Forum

Get expert LSAT preparation and law school admissions advice from PowerScore Test Preparation.

 Administrator
PowerScore Staff
  • PowerScore Staff
  • Posts: 8917
  • Joined: Feb 02, 2011
|
#23440
Complete Question Explanation

Flaw in the Reasoning-#%. The correct answer choice is (B)

In this dialogue, Sandy makes the reasonable argument that the best time to play the lottery is when the pot is the biggest. Alex asserts that it is better to play when the pot is lower and there are less people playing.
The flaw in Alex’s reasoning is that he seems to think of lottery players as competitors who can have an effect on each other’s odds of winning. The truth is that one’s odds of winning the lottery are the same regardless of how many other lottery players there are.

Answer choice (A): There is no referenced in Sandy’s argument to the number of times one plays, so this answer choice is incorrect.

Answer choice (B): This is the correct answer choice. Alex asserts that the number of other players has some effect on Sandy’s odds, which they do not.

Answer choice (C): Sandy does not assert that one’s chances are unaffected by the size of the pot; rather, the assertion is simply that it is better to play when the pot is bigger.

Answer choice (D): Alex does not make this assertion—his argument is based on the number of people playing the lottery in any given week.

Answer choice (E): Sandy doesn’t argue that the chances of winning go up under such circumstances, just that the pot increases after each time the lottery isn’t won.
 SherryZ
  • Posts: 124
  • Joined: Oct 06, 2013
|
#12764
Dec 2001 LSAT, Sec 3, Q 21:

I could not find any flaw when I saw this question (neither Sandy nor Alex). Could you explain this question and analyze each answer choices? :cry:

Thank you so much for your time!

---Sherry
 BethRibet
PowerScore Staff
  • PowerScore Staff
  • Posts: 200
  • Joined: Oct 17, 2012
|
#12792
Hi Sherry,

Thanks for your question.

The flaw in the reasoning shows up in Alex's response. Sandy explains that the Big Bucks lottery is such that multiple people can win it at the same time -- essentially everyone who chooses the five correct numbers.

Alex confuses the likelihood of winning, which is based only on whether or not you choose the right five numbers, with how large or small the pot is, depending on how many others are playing. But as the lottery is defined, your chances of winning are the same no matter how many other people are playing. The number of other players affects how much money you're likely to get, not how likely you are to win.

As far as the answer choices:
A) Sandy does not make any statement that matches what this answer choice claims.
B) This answer choice is correct, because it describes the flawed reasoning in Alex's response.
C) Whether or not Sandy does make this claim (if she does, it would be implicit, not clearly present), it is correct, not flawed.
D) This answer choice is inaccurate; Alex does not make this claim.
E) This answer choice is inaccurate; Sandy does not make this claim.

Hope this helps!
Beth

Get the most out of your LSAT Prep Plus subscription.

Analyze and track your performance with our Testing and Analytics Package.